The polynomial $x^6+x^3+1$ is irreducible over $mathbb{Q}[x]$












2












$begingroup$


To solve Dummit-Foote's exercise I want to check that whether the polynomial $p(x)= x^6+x^3+1in mathbb{Z}[x]$ is reducible over $mathbb{Q}$ or not.



I've seen that this polynomial has no zero in $mathbb{Q}$. But the degree of this polynomial is other than 2 and 3. Therefore, I can not say directly that this polynomial is irreducible over $mathbb{Q}$.



How do I show that the polynomial $x^6+x^3+1in mathbb{Z}[x]$ is irreducible over $mathbb{Q}$? Can anyone give me some hints?



Thank You in advance.










share|cite|improve this question











$endgroup$








  • 4




    $begingroup$
    Please add context to your question. Here are some tips on how to ask a good question.
    $endgroup$
    – André 3000
    Jan 31 at 16:29






  • 1




    $begingroup$
    Hey dude, that's three Problem-Statement-Questions asking for "show irreducible" in two hours. Before you carpet bomb us with any more, please go back and add context to the questions, or at least learn from the earlier responses before you pepper us with more. Thanks
    $endgroup$
    – rschwieb
    Jan 31 at 18:06












  • $begingroup$
    I'm really very sorry for my unexpected manner of questioning. I think I've edited my question properly now. Thanks every one for commenting here.
    $endgroup$
    – BijanDatta
    Feb 1 at 8:17
















2












$begingroup$


To solve Dummit-Foote's exercise I want to check that whether the polynomial $p(x)= x^6+x^3+1in mathbb{Z}[x]$ is reducible over $mathbb{Q}$ or not.



I've seen that this polynomial has no zero in $mathbb{Q}$. But the degree of this polynomial is other than 2 and 3. Therefore, I can not say directly that this polynomial is irreducible over $mathbb{Q}$.



How do I show that the polynomial $x^6+x^3+1in mathbb{Z}[x]$ is irreducible over $mathbb{Q}$? Can anyone give me some hints?



Thank You in advance.










share|cite|improve this question











$endgroup$








  • 4




    $begingroup$
    Please add context to your question. Here are some tips on how to ask a good question.
    $endgroup$
    – André 3000
    Jan 31 at 16:29






  • 1




    $begingroup$
    Hey dude, that's three Problem-Statement-Questions asking for "show irreducible" in two hours. Before you carpet bomb us with any more, please go back and add context to the questions, or at least learn from the earlier responses before you pepper us with more. Thanks
    $endgroup$
    – rschwieb
    Jan 31 at 18:06












  • $begingroup$
    I'm really very sorry for my unexpected manner of questioning. I think I've edited my question properly now. Thanks every one for commenting here.
    $endgroup$
    – BijanDatta
    Feb 1 at 8:17














2












2








2





$begingroup$


To solve Dummit-Foote's exercise I want to check that whether the polynomial $p(x)= x^6+x^3+1in mathbb{Z}[x]$ is reducible over $mathbb{Q}$ or not.



I've seen that this polynomial has no zero in $mathbb{Q}$. But the degree of this polynomial is other than 2 and 3. Therefore, I can not say directly that this polynomial is irreducible over $mathbb{Q}$.



How do I show that the polynomial $x^6+x^3+1in mathbb{Z}[x]$ is irreducible over $mathbb{Q}$? Can anyone give me some hints?



Thank You in advance.










share|cite|improve this question











$endgroup$




To solve Dummit-Foote's exercise I want to check that whether the polynomial $p(x)= x^6+x^3+1in mathbb{Z}[x]$ is reducible over $mathbb{Q}$ or not.



I've seen that this polynomial has no zero in $mathbb{Q}$. But the degree of this polynomial is other than 2 and 3. Therefore, I can not say directly that this polynomial is irreducible over $mathbb{Q}$.



How do I show that the polynomial $x^6+x^3+1in mathbb{Z}[x]$ is irreducible over $mathbb{Q}$? Can anyone give me some hints?



Thank You in advance.







abstract-algebra ring-theory field-theory irreducible-polynomials






share|cite|improve this question















share|cite|improve this question













share|cite|improve this question




share|cite|improve this question








edited Feb 1 at 8:11







BijanDatta

















asked Jan 31 at 16:21









BijanDattaBijanDatta

309113




309113








  • 4




    $begingroup$
    Please add context to your question. Here are some tips on how to ask a good question.
    $endgroup$
    – André 3000
    Jan 31 at 16:29






  • 1




    $begingroup$
    Hey dude, that's three Problem-Statement-Questions asking for "show irreducible" in two hours. Before you carpet bomb us with any more, please go back and add context to the questions, or at least learn from the earlier responses before you pepper us with more. Thanks
    $endgroup$
    – rschwieb
    Jan 31 at 18:06












  • $begingroup$
    I'm really very sorry for my unexpected manner of questioning. I think I've edited my question properly now. Thanks every one for commenting here.
    $endgroup$
    – BijanDatta
    Feb 1 at 8:17














  • 4




    $begingroup$
    Please add context to your question. Here are some tips on how to ask a good question.
    $endgroup$
    – André 3000
    Jan 31 at 16:29






  • 1




    $begingroup$
    Hey dude, that's three Problem-Statement-Questions asking for "show irreducible" in two hours. Before you carpet bomb us with any more, please go back and add context to the questions, or at least learn from the earlier responses before you pepper us with more. Thanks
    $endgroup$
    – rschwieb
    Jan 31 at 18:06












  • $begingroup$
    I'm really very sorry for my unexpected manner of questioning. I think I've edited my question properly now. Thanks every one for commenting here.
    $endgroup$
    – BijanDatta
    Feb 1 at 8:17








4




4




$begingroup$
Please add context to your question. Here are some tips on how to ask a good question.
$endgroup$
– André 3000
Jan 31 at 16:29




$begingroup$
Please add context to your question. Here are some tips on how to ask a good question.
$endgroup$
– André 3000
Jan 31 at 16:29




1




1




$begingroup$
Hey dude, that's three Problem-Statement-Questions asking for "show irreducible" in two hours. Before you carpet bomb us with any more, please go back and add context to the questions, or at least learn from the earlier responses before you pepper us with more. Thanks
$endgroup$
– rschwieb
Jan 31 at 18:06






$begingroup$
Hey dude, that's three Problem-Statement-Questions asking for "show irreducible" in two hours. Before you carpet bomb us with any more, please go back and add context to the questions, or at least learn from the earlier responses before you pepper us with more. Thanks
$endgroup$
– rschwieb
Jan 31 at 18:06














$begingroup$
I'm really very sorry for my unexpected manner of questioning. I think I've edited my question properly now. Thanks every one for commenting here.
$endgroup$
– BijanDatta
Feb 1 at 8:17




$begingroup$
I'm really very sorry for my unexpected manner of questioning. I think I've edited my question properly now. Thanks every one for commenting here.
$endgroup$
– BijanDatta
Feb 1 at 8:17










2 Answers
2






active

oldest

votes


















5












$begingroup$

I would do the following:




  1. Note that $p(x)$ is irreducible if and only if $p(x+1)$ is irreducible.


  2. Compute $p(x+1)$. We have
    begin{align}
    p(x+1)&=(x+1)^{6}+(x+1)^{3}+1\
    &=sum_{k=0}^{6}binom{6}{k}x^{k}+sum_{k=0}^{3}binom{3}{k}x^{k}+1\
    &=(x^{6}+6x^{5}+15x^{4}+20x^{3}+15x^{2}+6x+1)+(x^{3}+3x^{2}+3x+1)+1\
    &=x^{6}+6x^{5}+15x^{4}+21x^{3}+18x^{2}+9x+3
    end{align}

    Can you see that this polynomial is irreducible over $mathbb{Q}$?







share|cite|improve this answer









$endgroup$













  • $begingroup$
    Eisenstein criterion when p=3, right?
    $endgroup$
    – pendermath
    Feb 1 at 13:43










  • $begingroup$
    @pendermath Exactly
    $endgroup$
    – studiosus
    Feb 1 at 13:44










  • $begingroup$
    However, how did you come up with the substitution p(x+1)? There are many of these to prove (eg, 1, -1, 2, -2, etc.), and success is not even guaranteed.
    $endgroup$
    – pendermath
    Feb 1 at 13:45



















2












$begingroup$

studiosus answer has a much better argument, but in my experience my students always had trouble finding good substitutitions. So here is a harder, less elegant, but more staightforward way.



Note that it is an integer polynomial, so it is enough to decide irreduciblity in $mathbb{Z}[X]$ by one of the many results which claim the name 'Gauss Lemma'.



If $p$ were reducible over $mathbb{Z}$, then it would also be reducible after 'going mod 2', i.e. in $mathbb{F}_2[X]$. However in $mathbb{F}_2[X]$ it is easy to go through the different cases. Clearly $overline{p}$ has no roots, i.e. no factors of degree 1. The only irreducible polynomial of degree $2$ is $X^2+X+1$ and one can simply check that $p=(X^2+X+1)(X^4+X^3)+1$. Also there are just two irreducible polynomials of degree 3, namely $X^3+X+1$ and $X^3+X^2+1$. All possible products of those two come out to be different from $p$.



So $p$ does not have factors of degree 1, 2, or 3. So it must be irreducible in $mathbb{F}_2[X]$, hence in $mathbb{Z}[X]$, hence in $mathbb{Q}[X]$.






share|cite|improve this answer









$endgroup$














    Your Answer





    StackExchange.ifUsing("editor", function () {
    return StackExchange.using("mathjaxEditing", function () {
    StackExchange.MarkdownEditor.creationCallbacks.add(function (editor, postfix) {
    StackExchange.mathjaxEditing.prepareWmdForMathJax(editor, postfix, [["$", "$"], ["\\(","\\)"]]);
    });
    });
    }, "mathjax-editing");

    StackExchange.ready(function() {
    var channelOptions = {
    tags: "".split(" "),
    id: "69"
    };
    initTagRenderer("".split(" "), "".split(" "), channelOptions);

    StackExchange.using("externalEditor", function() {
    // Have to fire editor after snippets, if snippets enabled
    if (StackExchange.settings.snippets.snippetsEnabled) {
    StackExchange.using("snippets", function() {
    createEditor();
    });
    }
    else {
    createEditor();
    }
    });

    function createEditor() {
    StackExchange.prepareEditor({
    heartbeatType: 'answer',
    autoActivateHeartbeat: false,
    convertImagesToLinks: true,
    noModals: true,
    showLowRepImageUploadWarning: true,
    reputationToPostImages: 10,
    bindNavPrevention: true,
    postfix: "",
    imageUploader: {
    brandingHtml: "Powered by u003ca class="icon-imgur-white" href="https://imgur.com/"u003eu003c/au003e",
    contentPolicyHtml: "User contributions licensed under u003ca href="https://creativecommons.org/licenses/by-sa/3.0/"u003ecc by-sa 3.0 with attribution requiredu003c/au003e u003ca href="https://stackoverflow.com/legal/content-policy"u003e(content policy)u003c/au003e",
    allowUrls: true
    },
    noCode: true, onDemand: true,
    discardSelector: ".discard-answer"
    ,immediatelyShowMarkdownHelp:true
    });


    }
    });














    draft saved

    draft discarded


















    StackExchange.ready(
    function () {
    StackExchange.openid.initPostLogin('.new-post-login', 'https%3a%2f%2fmath.stackexchange.com%2fquestions%2f3095081%2fthe-polynomial-x6x31-is-irreducible-over-mathbbqx%23new-answer', 'question_page');
    }
    );

    Post as a guest















    Required, but never shown

























    2 Answers
    2






    active

    oldest

    votes








    2 Answers
    2






    active

    oldest

    votes









    active

    oldest

    votes






    active

    oldest

    votes









    5












    $begingroup$

    I would do the following:




    1. Note that $p(x)$ is irreducible if and only if $p(x+1)$ is irreducible.


    2. Compute $p(x+1)$. We have
      begin{align}
      p(x+1)&=(x+1)^{6}+(x+1)^{3}+1\
      &=sum_{k=0}^{6}binom{6}{k}x^{k}+sum_{k=0}^{3}binom{3}{k}x^{k}+1\
      &=(x^{6}+6x^{5}+15x^{4}+20x^{3}+15x^{2}+6x+1)+(x^{3}+3x^{2}+3x+1)+1\
      &=x^{6}+6x^{5}+15x^{4}+21x^{3}+18x^{2}+9x+3
      end{align}

      Can you see that this polynomial is irreducible over $mathbb{Q}$?







    share|cite|improve this answer









    $endgroup$













    • $begingroup$
      Eisenstein criterion when p=3, right?
      $endgroup$
      – pendermath
      Feb 1 at 13:43










    • $begingroup$
      @pendermath Exactly
      $endgroup$
      – studiosus
      Feb 1 at 13:44










    • $begingroup$
      However, how did you come up with the substitution p(x+1)? There are many of these to prove (eg, 1, -1, 2, -2, etc.), and success is not even guaranteed.
      $endgroup$
      – pendermath
      Feb 1 at 13:45
















    5












    $begingroup$

    I would do the following:




    1. Note that $p(x)$ is irreducible if and only if $p(x+1)$ is irreducible.


    2. Compute $p(x+1)$. We have
      begin{align}
      p(x+1)&=(x+1)^{6}+(x+1)^{3}+1\
      &=sum_{k=0}^{6}binom{6}{k}x^{k}+sum_{k=0}^{3}binom{3}{k}x^{k}+1\
      &=(x^{6}+6x^{5}+15x^{4}+20x^{3}+15x^{2}+6x+1)+(x^{3}+3x^{2}+3x+1)+1\
      &=x^{6}+6x^{5}+15x^{4}+21x^{3}+18x^{2}+9x+3
      end{align}

      Can you see that this polynomial is irreducible over $mathbb{Q}$?







    share|cite|improve this answer









    $endgroup$













    • $begingroup$
      Eisenstein criterion when p=3, right?
      $endgroup$
      – pendermath
      Feb 1 at 13:43










    • $begingroup$
      @pendermath Exactly
      $endgroup$
      – studiosus
      Feb 1 at 13:44










    • $begingroup$
      However, how did you come up with the substitution p(x+1)? There are many of these to prove (eg, 1, -1, 2, -2, etc.), and success is not even guaranteed.
      $endgroup$
      – pendermath
      Feb 1 at 13:45














    5












    5








    5





    $begingroup$

    I would do the following:




    1. Note that $p(x)$ is irreducible if and only if $p(x+1)$ is irreducible.


    2. Compute $p(x+1)$. We have
      begin{align}
      p(x+1)&=(x+1)^{6}+(x+1)^{3}+1\
      &=sum_{k=0}^{6}binom{6}{k}x^{k}+sum_{k=0}^{3}binom{3}{k}x^{k}+1\
      &=(x^{6}+6x^{5}+15x^{4}+20x^{3}+15x^{2}+6x+1)+(x^{3}+3x^{2}+3x+1)+1\
      &=x^{6}+6x^{5}+15x^{4}+21x^{3}+18x^{2}+9x+3
      end{align}

      Can you see that this polynomial is irreducible over $mathbb{Q}$?







    share|cite|improve this answer









    $endgroup$



    I would do the following:




    1. Note that $p(x)$ is irreducible if and only if $p(x+1)$ is irreducible.


    2. Compute $p(x+1)$. We have
      begin{align}
      p(x+1)&=(x+1)^{6}+(x+1)^{3}+1\
      &=sum_{k=0}^{6}binom{6}{k}x^{k}+sum_{k=0}^{3}binom{3}{k}x^{k}+1\
      &=(x^{6}+6x^{5}+15x^{4}+20x^{3}+15x^{2}+6x+1)+(x^{3}+3x^{2}+3x+1)+1\
      &=x^{6}+6x^{5}+15x^{4}+21x^{3}+18x^{2}+9x+3
      end{align}

      Can you see that this polynomial is irreducible over $mathbb{Q}$?








    share|cite|improve this answer












    share|cite|improve this answer



    share|cite|improve this answer










    answered Feb 1 at 12:58









    studiosusstudiosus

    2,294815




    2,294815












    • $begingroup$
      Eisenstein criterion when p=3, right?
      $endgroup$
      – pendermath
      Feb 1 at 13:43










    • $begingroup$
      @pendermath Exactly
      $endgroup$
      – studiosus
      Feb 1 at 13:44










    • $begingroup$
      However, how did you come up with the substitution p(x+1)? There are many of these to prove (eg, 1, -1, 2, -2, etc.), and success is not even guaranteed.
      $endgroup$
      – pendermath
      Feb 1 at 13:45


















    • $begingroup$
      Eisenstein criterion when p=3, right?
      $endgroup$
      – pendermath
      Feb 1 at 13:43










    • $begingroup$
      @pendermath Exactly
      $endgroup$
      – studiosus
      Feb 1 at 13:44










    • $begingroup$
      However, how did you come up with the substitution p(x+1)? There are many of these to prove (eg, 1, -1, 2, -2, etc.), and success is not even guaranteed.
      $endgroup$
      – pendermath
      Feb 1 at 13:45
















    $begingroup$
    Eisenstein criterion when p=3, right?
    $endgroup$
    – pendermath
    Feb 1 at 13:43




    $begingroup$
    Eisenstein criterion when p=3, right?
    $endgroup$
    – pendermath
    Feb 1 at 13:43












    $begingroup$
    @pendermath Exactly
    $endgroup$
    – studiosus
    Feb 1 at 13:44




    $begingroup$
    @pendermath Exactly
    $endgroup$
    – studiosus
    Feb 1 at 13:44












    $begingroup$
    However, how did you come up with the substitution p(x+1)? There are many of these to prove (eg, 1, -1, 2, -2, etc.), and success is not even guaranteed.
    $endgroup$
    – pendermath
    Feb 1 at 13:45




    $begingroup$
    However, how did you come up with the substitution p(x+1)? There are many of these to prove (eg, 1, -1, 2, -2, etc.), and success is not even guaranteed.
    $endgroup$
    – pendermath
    Feb 1 at 13:45











    2












    $begingroup$

    studiosus answer has a much better argument, but in my experience my students always had trouble finding good substitutitions. So here is a harder, less elegant, but more staightforward way.



    Note that it is an integer polynomial, so it is enough to decide irreduciblity in $mathbb{Z}[X]$ by one of the many results which claim the name 'Gauss Lemma'.



    If $p$ were reducible over $mathbb{Z}$, then it would also be reducible after 'going mod 2', i.e. in $mathbb{F}_2[X]$. However in $mathbb{F}_2[X]$ it is easy to go through the different cases. Clearly $overline{p}$ has no roots, i.e. no factors of degree 1. The only irreducible polynomial of degree $2$ is $X^2+X+1$ and one can simply check that $p=(X^2+X+1)(X^4+X^3)+1$. Also there are just two irreducible polynomials of degree 3, namely $X^3+X+1$ and $X^3+X^2+1$. All possible products of those two come out to be different from $p$.



    So $p$ does not have factors of degree 1, 2, or 3. So it must be irreducible in $mathbb{F}_2[X]$, hence in $mathbb{Z}[X]$, hence in $mathbb{Q}[X]$.






    share|cite|improve this answer









    $endgroup$


















      2












      $begingroup$

      studiosus answer has a much better argument, but in my experience my students always had trouble finding good substitutitions. So here is a harder, less elegant, but more staightforward way.



      Note that it is an integer polynomial, so it is enough to decide irreduciblity in $mathbb{Z}[X]$ by one of the many results which claim the name 'Gauss Lemma'.



      If $p$ were reducible over $mathbb{Z}$, then it would also be reducible after 'going mod 2', i.e. in $mathbb{F}_2[X]$. However in $mathbb{F}_2[X]$ it is easy to go through the different cases. Clearly $overline{p}$ has no roots, i.e. no factors of degree 1. The only irreducible polynomial of degree $2$ is $X^2+X+1$ and one can simply check that $p=(X^2+X+1)(X^4+X^3)+1$. Also there are just two irreducible polynomials of degree 3, namely $X^3+X+1$ and $X^3+X^2+1$. All possible products of those two come out to be different from $p$.



      So $p$ does not have factors of degree 1, 2, or 3. So it must be irreducible in $mathbb{F}_2[X]$, hence in $mathbb{Z}[X]$, hence in $mathbb{Q}[X]$.






      share|cite|improve this answer









      $endgroup$
















        2












        2








        2





        $begingroup$

        studiosus answer has a much better argument, but in my experience my students always had trouble finding good substitutitions. So here is a harder, less elegant, but more staightforward way.



        Note that it is an integer polynomial, so it is enough to decide irreduciblity in $mathbb{Z}[X]$ by one of the many results which claim the name 'Gauss Lemma'.



        If $p$ were reducible over $mathbb{Z}$, then it would also be reducible after 'going mod 2', i.e. in $mathbb{F}_2[X]$. However in $mathbb{F}_2[X]$ it is easy to go through the different cases. Clearly $overline{p}$ has no roots, i.e. no factors of degree 1. The only irreducible polynomial of degree $2$ is $X^2+X+1$ and one can simply check that $p=(X^2+X+1)(X^4+X^3)+1$. Also there are just two irreducible polynomials of degree 3, namely $X^3+X+1$ and $X^3+X^2+1$. All possible products of those two come out to be different from $p$.



        So $p$ does not have factors of degree 1, 2, or 3. So it must be irreducible in $mathbb{F}_2[X]$, hence in $mathbb{Z}[X]$, hence in $mathbb{Q}[X]$.






        share|cite|improve this answer









        $endgroup$



        studiosus answer has a much better argument, but in my experience my students always had trouble finding good substitutitions. So here is a harder, less elegant, but more staightforward way.



        Note that it is an integer polynomial, so it is enough to decide irreduciblity in $mathbb{Z}[X]$ by one of the many results which claim the name 'Gauss Lemma'.



        If $p$ were reducible over $mathbb{Z}$, then it would also be reducible after 'going mod 2', i.e. in $mathbb{F}_2[X]$. However in $mathbb{F}_2[X]$ it is easy to go through the different cases. Clearly $overline{p}$ has no roots, i.e. no factors of degree 1. The only irreducible polynomial of degree $2$ is $X^2+X+1$ and one can simply check that $p=(X^2+X+1)(X^4+X^3)+1$. Also there are just two irreducible polynomials of degree 3, namely $X^3+X+1$ and $X^3+X^2+1$. All possible products of those two come out to be different from $p$.



        So $p$ does not have factors of degree 1, 2, or 3. So it must be irreducible in $mathbb{F}_2[X]$, hence in $mathbb{Z}[X]$, hence in $mathbb{Q}[X]$.







        share|cite|improve this answer












        share|cite|improve this answer



        share|cite|improve this answer










        answered Feb 1 at 13:38









        Rene RecktenwaldRene Recktenwald

        1808




        1808






























            draft saved

            draft discarded




















































            Thanks for contributing an answer to Mathematics Stack Exchange!


            • Please be sure to answer the question. Provide details and share your research!

            But avoid



            • Asking for help, clarification, or responding to other answers.

            • Making statements based on opinion; back them up with references or personal experience.


            Use MathJax to format equations. MathJax reference.


            To learn more, see our tips on writing great answers.




            draft saved


            draft discarded














            StackExchange.ready(
            function () {
            StackExchange.openid.initPostLogin('.new-post-login', 'https%3a%2f%2fmath.stackexchange.com%2fquestions%2f3095081%2fthe-polynomial-x6x31-is-irreducible-over-mathbbqx%23new-answer', 'question_page');
            }
            );

            Post as a guest















            Required, but never shown





















































            Required, but never shown














            Required, but never shown












            Required, but never shown







            Required, but never shown

































            Required, but never shown














            Required, but never shown












            Required, but never shown







            Required, but never shown







            Popular posts from this blog

            Can a sorcerer learn a 5th-level spell early by creating spell slots using the Font of Magic feature?

            Does disintegrating a polymorphed enemy still kill it after the 2018 errata?

            A Topological Invariant for $pi_3(U(n))$